{\frac{1}{2}}, $$

Так как \(0 \lt \frac{1}{5} \lt 1\) и \(\frac{1}{3} \lt \frac{1}{2}\).

Тест по теме «Степени с рациональными показателями и их свойства»

Методическая разработка «Контрольная работа в формате теста»

Тема «Степени с рациональным показателем и их свойства»

Тест предназначен для проведения контроля знаний учащихся 10 класса или студентов 1 курса СПО в конце изучения темы «Степени с рациональным показателем и их свойства». Тест дает возможность быстро и эффективно провести диагностику усвоения материала по теме. Данный вид контроля стимулирует у учащихся стремление к систематической самостоятельной работе по изучению дисциплины.

Цель тестирования: тематический контроль.

Общее время на выполнение работы: 30-40 минут.

Структура работы и типы заданий. Контрольно-измерительный материал (тест) представлен двумя эквивалентными по содержанию и сложности вариантам, каждый из которых состоит из 8 заданий (задания № 1, 3, 4 — закрытое задание с выбором одного правильного ответа, задание № 2 – задание на восстановление соответствия, задание № 5 – задание на установление последовательности; № 6 – открытое задание с кратким ответом, № 7 – закрытое задание с выбором нескольких правильных ответов, № 8 – открытое задание с кратким ответом).

Вариант 1

1. Упростите

а)

б)

в)

г)

2. Установите соответствие между выражениями и их значениями.

А) ;

Б) ;

В) ;

Г) ;

Д) .

1) 2;

2) 4;

3) 10;

4) 0,125;

5) 8.

3. Найдите значение выражения: при

а) 5;

б) 25;

в) 0,2;

г) .

4. Выполните действия:

а) ;

б) ;

в) ;

г) .

5. Запишите числа в порядке возрастания ; ; ; .

а) ; ; ; .

б) ; ; ; .

в) ; ; ; .

г) ; ; ; .

6. Даны числа: . Проанализируйте их и заполните пропуски:

Среди указанных чисел наименьшим является число _____, оно равно ______.  А наибольшим является число ______, оно равно _______.

7. Упростите выражение:

1) 2) 3) 4)

и найдите его значение, если :

а) 256; б) 1024; в) ; г) .

8. Вычислите и запишите ответ __________.

Вариант 2

1. Упростите

а)

б)

в)

г)

2. Установите соответствие между выражениями и их значениями.

А) ;

Б) ;

В) ;

Г) ;

Д)

1) 3;

2) 27;

3) 18;

4) 0,2;

5) 9.

3. Найдите значение выражения: при

а) 7;

б) ;

в) 49;

г) .

4. Выполните действия:

а) ;

б) ;

в) ;

г) .

5. Запишите числа в порядке убывания ; ; ; .

а) ; ; ; .

б) ; ; ; .

в) ; ; ; .

г) ; ; ; .

6. Даны числа: . Проанализируйте их и заполните пропуски:

Среди указанных чисел наименьшим является число _____, оно равно ______. А наибольшим является число ______, оно равно _______.

7. Упростите выражение:

1) 2) 3) 4)

и найдите его значение, если :

а) 40,5; б) 24,3; в) 121,5; г) 243.

8. Вычислите и запишите ответ __________.

реальный анализ — Покажите, что f прерывисто в каждом рациональном и непрерывном в каждом иррациональном.

Эта функция называется функцией Тома (наряду с множеством других названий). На его странице в Википедии есть неофициальное доказательство того, что он непрерывен в иррациональных числах и прерывист в рациональных числах

.

Ясно, что $f$ разрывна во всех рациональных числах: поскольку иррациональные числа плотны в вещественных числах, для любого рационального $x$, независимо от того, какой $\epsilon$ мы выберем, найдется иррациональное a, еще более близкое к нашему $x $ где $f(a) = 0$ (при этом $f(x)$ положительно). Другими словами, $f$ никогда не может «приблизиться» и «остаться близким» ни к одному положительному числу, потому что его область определения заполнена нулями. Чтобы показать непрерывность в иррациональных числах, предположим без ограничения общности, что наш $\epsilon$ является рациональным (для любого иррационального $\epsilon’$ мы можем выбрать меньшее рациональное $\epsilon»$, и доказательство транзитивно). Поскольку $\epsilon$ рационально, его можно выразить в самых низких терминах как $\frac{a}{b}$. Мы хотим показать, что $f(x)$ непрерывна, когда $x$ иррационально. Обратите внимание, что $f$ принимает максимальное значение $1$ для каждого целого числа, поэтому мы можем ограничить наше исследование пространством между $\lfloor x\rfloor$ и $\lceil x\rceil$. Поскольку $\epsilon$ имеет конечный знаменатель, равный $b$, единственными значениями, для которых $f$ может возвращать значение больше $\epsilon$, являются те, у которых сокращенный знаменатель не превышает $b$. Между двумя целыми числами со знаменателем, не превышающим $b$, существует только конечное число значений, поэтому их можно перечислить исчерпывающе.

Установка $\delta$ меньше ближайшего расстояния от $x$ до одного из этих значений гарантирует, что каждое значение в пределах $\delta$ от $x$ имеет $f(x) < \epsilon$.

Если вам нужно освежить знания о плотных наборах, посетите страницу википедии, озаглавленную «Dense Set».

Доказательство плотности рациональных чисел в $\mathbb{R}$

Доказательство плотности иррациональных чисел в $\mathbb{R}$ (см. первый ответ, а не вопрос) показ функции не является непрерывной справа показывает, что она не является непрерывной, поэтому вы можете пропустить последние шаги (с рациональным пределом слева), содержащие $\lim\limits_{m \rightarrow \infty} f(x_m) = \ lim\limits_{m \rightarrow \infty} \frac{pm-q}{qm}=\frac{1}{qm} \neq f(x)=\frac{1}{q}$. Во-вторых, $\lim\limits_{n \rightarrow \infty} f(x_n) = \lim\limits_{n \rightarrow \infty} \frac{pn+q}{qn}= \lim\limits_{n \rightarrow \ infty} \frac{p+\frac{q}{n}}{q} = \frac{p}{q} \neq f(x)=\frac{1}{q}$ Но хотя $p$ обычно не $1$, может быть, поэтому я думаю, что вы должны подойти к этому пределу с точки зрения реальных чисел, а не только рациональных.

Наконец, вам действительно нужно больше обосновать первую строку. Поскольку рациональные числа плотны в $\mathbb{R}$, не должно ли это означать, что в любом интервале вокруг любого иррационального числа всегда есть рациональное число? Если это так, то значение $f$ в этой точке будет ненулевым, так что дело в непрерывности выглядит плохо. Большая часть цитируемого Википедии плана доказательства развеивает этот страх. Он делает это, показывая для любого произвольного положительного $\epsilon$, если $c$ находится в пределах четко определенной $\delta$ любого иррационального числа, назовем его x $|f(x+c)| < \эпсилон$. Говоря более кратко, они применяют эпсилон-дельта определение непрерывности.

Надеюсь, это поможет вам лучше интуитивно понять, что функция Дирихле (или индикаторная функция рациональных чисел) нигде не является непрерывной. Это если функция $g : g(x) = 1 \text{ if } x \in \mathbb{Q}\ \text{ иначе }g(x) = 0$. Есть неофициальное доказательство того, что оно прерывисто везде на этой странице Википедии, но это прямой результат плотности как рациональных, так и иррациональных чисел в $\mathbb{R}$.

реальный анализ — Является ли индикаторная функция рациональных чисел интегрируемой по Риману?

Задавать вопрос

спросил

8 лет, 2 месяца назад

Изменено 7 месяцев назад

Просмотрено 10 тысяч раз

$\begingroup$

$f(x) = \begin{случаи} 1 & х\in\Bbb Q \\[2ex] 0 & x\notin\Bbb Q \end{case}$ 9я}]$. Таким образом, все рациональные числа из $[0,1]$ можно покрыть множеством меры $\epsilon$. На этом наборе верхняя сумма равна $1\times\epsilon=\epsilon$. Из этого набора верхняя сумма равна 0. Таким образом, верхняя сумма и нижняя сумма отличаются на любой произвольный $\epsilon$. Таким образом, функция интегрируема.

Один из приведенных выше аргументов должен быть неправильным. Пожалуйста, дайте мне знать, какой из них неправильный и почему. Любая помощь приветствуется.

  • реальный анализ
  • интегрирование
  • анализ
  • определенные интегралы
  • сумма Римана

$\endgroup$

5

$\begingroup$

Ответ зависит от того, в каком смысле вы хотите интегрировать функцию.

Функция не интегрируема по Риману. Проблема в том, что вы считаете, что конечные разбиения образуют суммы Римана; так что, грубо говоря, вы не можете сделать выбор для каждого рационального, рассматривая конечные разделы. (Ваш первый аргумент верен.)

Однако функция интегрируема по Лебегу. Здесь аргумент с использованием меры уместен.

Функция также интегрируема в смысле Хенстока-Курцвейла. Грубо говоря, это похоже на Римана, но на самом деле позволяет сделать «больше выбора».

$\endgroup$

$\begingroup$

Напомним, что функция интегрируема по Риману тогда и только тогда, когда для любого $\varepsilon >

0$ существует разбиение $P$ такое, что $U(f, P) — L(f, P) < \varepsilon$ . 9n (x_k - x_{k-1}) = 1$$

Таким образом, критерий интегрируемости в первой строке не выполняется для любого $\varepsilon < 1$.

$\endgroup$

$\begingroup$

Важная теорема об интегрируемости по Риману известна как Критерий Лебега . Он гласит, что если у нас есть функция $f(x)$, определенная на отрезке $I$, то $f$ интегрируема на $I$ тогда и только тогда, когда $f$ ограничена и множество разрывов $f (x)$ имеет нулевую меру.

В этом случае множеством разрывов будет весь интервал $[0,1]$, поэтому по критерию Лебега функция неинтегрируема.

Определение «нулевой меры» тесно связано с вашим ошибочным доказательством: множество имеет нулевую меру, если для каждого $\epsilon > 0$ множество может быть покрыто последовательностью открытых интервалов, таких что сумма ширин открытых интервалов меньше $\epsilon$.